CPR I - First Aid Flashcards Preview

CPR I > CPR I - First Aid > Flashcards

Flashcards in CPR I - First Aid Deck (149)
Loading flashcards...
1
Q

An intoxicated 25 year old student awakes the morning after the final examinations. While lying supine, he vomits and aspirates stomach contents. Several days later, after developing a high fever and coughing purulent sputum, he seeks medical attention. The chest X-Rays demonstrate an early consolidating pneumonia. The region most likely involved is the ______?

A

Superior segment, right lower lobe. In the supine position, the most gravity dependent segmental bronchus is the superior segment of the right inferior lobe. In this case, it is therefore the most likely site of aspiration pneumonia.

2
Q

A 23 year-old male is brought to your emergency room following a stab wound. Anatomically, you note a well-demarcated, horizontal, 5 cm laceration just to right of the sternum, at approximately 6 centimeters below the sternal angle (of Louis). His findings include: blood pressure 72/40; heart rate 120/min, regular; distended neck veins; clear lung fields via auscultation; and muffled (distant) heart sounds (so-called “Beck’s Triad”). Due to the location of the wound, you suspect a _______?

A

Cardiac Tamponade. In the presence of bleeding from a stab wound to the heart, pressure around the heart gradually increases as blood leaks from the affected cardiac chamber. This increasing pressure eventually prevents the heart from filling normally and cardiac output falls (hence the drop in blood pressure and cardiac output). The pressure backs up, causing the large veins to dilate (distention of the neck veins), and the increasing layer of blood in the pericardium muffles the heart sounds. This can rapidly cause acute cardiac arrest and death.

3
Q

You have given a patient a new drug whose only effect is that it prevents only the funny sodium channels (if) in the heart from opening. What effect will this drug have on the heart rate?

A

Funny Na+ channels help depolarize the nodal action potential to threshold. One consequence of blocking these channels would be to slow the rise of the nodal pacemaker potential toward threshold. This would increase the interval between nodal action potentials and slow the heart.

4
Q

A 77 year old male with a history of heart failure comes in today and on his electrocardiogram shows atrial fibrillation (the atria are not contracting). On prior exams he has had a normal S1, normally split S2 and an S3 and S4. You note that one of these heart sounds is missing. Which one?

A

S4 is the heart sound produced by the contraction of the atria usually found in somewhat noncompliant (stiff hearts in patients with hypertension). In this case however the patient is in atrial fibrillation in which the atria are fibrillating and there is no organized atrial contraction. Hence the S4 will be absent in patients with atrial fibrillation.

5
Q

Your patient starts to take a new medication that has a side-effect to inhibit cardiac voltage dependent potassium channels (IKr and IKs). What change would you expect to observe in the ECG?

A

Prolongation of the QTc interval. The QT interval represents repolarization of the ventricles. Ventricular repolarization is controlled by the rapidly (IKr) and slowly (IKs) activating delayed rectifier potassium channels. Decreases in activity of either channel will prolong the QT interval.

6
Q

Drug X is…

A

Metoprolol. Metoprolol is a beta-1 selective antagonist. In the figure on left, Epi (apparently a low dose) is causing an increase in HR (beta-1 effect), a drop in diastolic BP (a beta-2 effect), and an increase in pulse pressure (the difference between diastolic and systolic pressure that is due mostly to beta-1 stimulated increase in HR and contractile force). After drug X is given, Epi no longer stimulates HR (beta-1 effect), and Epi no longer increases the pulse pressure (also a beta-1 effect). However, Epi continues to cause a drop in the diastolic pressure (a beta-2 effect). So, drug X must be a selective beta-1 antagonist, and metoprolol is the only beta-1 selective antagonist.

7
Q

Evaluate the mean QRS axis in your patient presenting with the following ECG.

A

Determining the mean QRS axis from an EKG requires measurements in two leads. Looking at lead I, the mean QRS axis is positive. If we look at lead aVF, the mean QRS axis is negative. Referencing the hexaxial diagram, a positive mean QRS axis in lead I and a negative mean QRS axis in lead aVF would fall between 0 and -90 degrees.

8
Q

Mr Jones comes to the Emergency Department with chest pain and shortness of breath. His 12 lead ECG and blood enzyme levels show an acute infarction in the inferior, posterior area of the left ventricle. Cardiac catheterization reveals a clot. His hemodynamic parameters are listed below. After a clot dissolving drug is infused into the coronary artery, the hemodynamics are shown below. Systemic vascular resistance (total peripheral vascular resistance) when he arrives is:

A

SVR = (mean arterial pressure – right atrial pressure) / Cardiac Output. (70 – 10)/3 = 60/3 = 20

9
Q

A mother presents with her 1 week old boy to your clinic. The baby was cyanotic at birth with a heart murmur. After further examination you determine that the child already has heart failure. What embryological problems might this child have with blood flow in his heart?

A

During normal development, neural crest differentiate into the truncus arteriosus and spiral into the aorticopulmonary septum. When it does not spiral, you get TRANSPOSITION of the great vessels. When the atrioventricular septum does not develop properly, you get TETRALOGY OF FALLOT and/or PERSISTENT TRUNCUS ARTERIOSUS.

10
Q

What embryonic structure gives rise to the outflow tract of the left and right ventricles?

A

Bulbus cordis

11
Q

You are performing an intrauterine surgery on a neonate due to heart problems. During the surgery, you note that the ventricles and atria lack trabeculae. What embryonic structes may have a mutation that would cause this?

A

The primitive ventricles and atria.

12
Q

You are performing an intrauterine surgery on a neonate that has had irregular heart beats. When doing your surgery, you notice that venous flow from the muscles of the heart is obstructed and the heart beats weakly. What embryonic structure may have a mutation in this child?

A

Left horn of the sinus venosus. This structure gives rise to the coronary sinus and the SA node. This explains decreased venous flow from the coronary vessels and irregular heart beat because of SA node dysfunction.

13
Q

What embryological structure forms the smooth part of the right atrium?

A

The right horn of the sinus venosus.

14
Q

A patient presents with with decreased cardiac output. EKG shows decreased p-wave and angiogram reveals a narrowed superior vena cava. What embryological structures were likely mutated in this patient?

A

Right common cardinal vein and right anterior cardinal vein

15
Q

How does the interventricular septum develop?

A
  • Muscular Ventricular Septum develops
  • Aorticopulmonary Septum spirals to form membranous interventricular septum
  • Endocardial Cushions fuse to membranous interventricular septum
16
Q

When dissecting your cadaver, you notice a spot between the left and right atria that is translucent to light. What normally happens during fetal development that makes that portion continuous with surrounding tissue?

A
  • Foramen primum narrows as septum primum grows toward endocardial cushions
  • Perforations in septum primum forms foramen secundum and the foramen primum disappears
  • Septum secundum grows and upper part of septum primum degenerates
  • Septum primum forms the valve of the foramen ovale
  • Septum secundum and septum primum fuse to form the atrial septum
  • Foramen ovale closes after birth from increase in left atrial pressure.
17
Q

Young neonates do not have bone marrow until about week 20. Before this, in what locations do neonates carry out hematopoiesis?

A

Yolk sac (3-10 wks). Liver (6 wks - birth). Spleen (15-30 wks). Bone Marrow (22 wks on). “Young Liver Synthesizes Blood”

18
Q

You invent a new medical device that can measure neonatal blood pressure. You test out your device and it tells you that the neonate has a higher pressure on the right side of her heart. Should you scrap the device or is this reading likely correct?

A

The reading is correct. Fetal circulation starts at the umbilical vein, traveling through the ductus venosus, bypassing the liver and going to the heart. Once blood gets to the heart, it is shunted through the ductus arteriosus if it needs to go back to the mother for more oxygen, or through the foramen ovale if it is to be pumped out into circulation. Because of this system, the right heart has more pressure in it.

19
Q

What are the post-natal derivatives of the following?

  • Umbilical vein
  • Umbilical arteries
  • Ductus arteriosus
  • Ductues venosus
  • Foramen ovale
  • Allantois
  • Notocord
A

Umbilical vein = ligamentum teres hepatis
Umbilical arteries = medial umbilical ligaments
Ductus arteriosus = ligamentum arteriosum
Ductues venosus = ligamentum venosum
Foramen ovale = fossa ovalis
Allantois = urachus-median umbilical ligament
Notocord = nucleus pulposus

20
Q

A 44 year old obese male presents to your clinic complaining of hoarseness and dysphagia (difficulty swallowing). What cardiac issue could be causing this patient’s symptoms?

A

Left atrail enlargment. Compression of the esophagus (dysphagia) and left recurrent laryngeal nerve (hoarseness) by the left atrium causes these symptoms.

21
Q

A patient comes to see you with chest pain. On his EKG you note problems in SA and AV node conduction. This patient also has very high cholesterol. How could the cholesterol of this patient be the cause of the findings in the patient’s EKG?

A

The right coronary artery provides blood supply for the SA and AV nodes.

22
Q

After experiencing mild chest pain and discomfort for the past few months, a patient has a heart attack. What is the most common artery that causes this patient’s condition?

A

The LAD (left anterior decending) artery is the most common coronary artery occlusion.

23
Q

What determines coronary circulation dominance?

A

Which artery supplies the posterior decending artery.

  • 85% of people have supply from the RCA (right coronary artery) = right-dominant circulation
  • 8% of people have supply from the LCX (left coronary circumflex artery) = left-dominant circulation
  • 7% of people have supply from the RCA and LCX = codominant circulation
24
Q

You are performing an angiogram on one of your patients to find the source of coronary artery blockage. When you squirt dye into the catheter, you see the dye run through and outline the coronary arteries. At what point in the cardiac cycle is the heart when you see the dye in the coronary arteries?

A

Diastole. This is when the coronary arteries fill with blood.

25
Q

Where do lung alveoli form from in the embryo?

A

Endoderm. They bud off of the foregut.

26
Q

A 33 year old patient comes to the ER complaining of chest pain, rapid palpitations and dizziness. She drinks four cups of coffee a day and 2 glasses of wine at night. ECG after the episode reveals paroxysmal ventricular tachycardia. You try the vagal maneuver to terminate the tachycardia, but it doesn’t work. What is your first drug of choice to control her PSVT?

A

Adenosine. It slows conduction at the AV node, slowing action potential initiation in the ventricles.

27
Q

Calculate vascular resistance if cardiac output is 5L/min, MAP is 108mmHg, left atrial pressure is 12mmHg, right atrial pressure is 8mmHg, stroke volume is 100mL and heart rate is 50 bpm.

A

Vascular resistance is equal to = change in pressure / flow. (MAP - right atrial pressure) / flow. (108 - 8) / 5 = 20.

28
Q

If a system filled with water has an outflow of 6 L/min, what would outflow of this sytem be if water was replaced by whole blood with a hematocrit reading of 45?

A

As hematocrit goes up so does viscosity. Viscosity of blood is about 3-4x that of water. Dividing 6 by 3 = 2.

29
Q

Mr. Jones comes to the ER with chest pain. His 12 lead ECG and blood enzyme levels show an acute infarction in the posterior inferior portion of the left ventricle. Cardiac catheterization shows a clot. Hemodynamic parameters are shown below when he arrived. Systemic vascular resistance (total peripheral resistance) when he arrives is what?

CO = 3

MAP = 70

Mean Pulmonary Artery Pressure = 24

Right Atrial Pressure = 10

Left Atrial Pressure = 12

Coronary Blood Flow = 10

A
  1. R = change in pressure/ flow. (MAP - right atrial pressure)/ CO. (70-10)/3 = 20.
30
Q

A previous patient of yours comes to the ER in atrial fibrillation. Previously he had all four heart sounds. Which heart sound will be absent when he comes to the ER in a-fib?

A

S4 because the atria are no longer contracting. S4 is a sound of stiff atrial valves kicking open.

31
Q

Which of the following drugs could be safely used to treat atrial fibrillation in someone with WPW syndrome?

  1. Beta blocker
  2. Amiodarone
  3. Digoxin
  4. Adenosine
  5. Verapamil
A

Amiodarone. In WPW the accessory pathway has Na+ channels you could target to specifically slow conduction through the accessory pathway. Amiodarone blocks Na+ channels as well as K+ channels. You wouldn’t want to use the other drugs because blocking the AV node would make the situation worse.

32
Q

A patient with WPW comes to the ER with chest pain and an irregular HR of 200 bpm. EKG shows a wide QRS complex. What does this patient likely have?

A

Atrial fibrillation

33
Q

In the image below, identify the left pulmonary artery.

A

First find the circular lucency which is the left main bronchus. Posterior to that is the left pulmonary artery (“Left overs”, anterior the right pulmonary artery and most superiorly is the aortic knob.

34
Q

A patient comes to the ED with crushing chest pain radiating down his left arm. He has a history of hypertrophic cardiomyopathy, but you determine that he is having an MI due to coronary occlusion. Why is nitroglycerine a poor choice of drug for this patient?

A

Nitroglycerine is an organic nitrate that vasodilates arteries and veins. This will decrease venous return to the heart and exacerbate the effects of the hypertrophic heart.

35
Q

A patient comes to your clinic with hypertension. He is an elderly, African-American man with low renin. What drugs are commonly effective in patients within his demographics? What if he had heart failure?

A

Calcium channel blockers. If he had heart failure, hydralazine has proven effective in African-American communities.

36
Q

A 51 year old male comes to see you because he has to keep taking sublingual organic nitrates for chest pain and you told him not to take them often or he will become tolerant to them. What drug could you combine with his current medication to reduce tolerance?

A

Hydralazine. (BiDil is isosorbide dinitrate and hydralazine combined)

37
Q

A patient comes to the ED with crushing chest pain and pain down his left arm. You have two drugs available to dilate his coronary arteries: nitroglycerin and sodium nitroprusside. Which one would you give if you wanted the fastest results? Why might the slower drug be a better choice?

A

Sodium nitroprusside. Though they both release NO to vasodilate, sodium nitroprusside releases it directly and does not need to be metabolized by the liver. Nitroglycerin might be a better choice because it specifically vasodilates the coronary vessels where sodium nitroprusside vasodilates everything and you can get “steal” of blood from coronary vessels.

38
Q

Which Wiggers diagram below shows aortic stenosis? Aortic regurgitation? Mirtal stenosis? Mitral regurgitation?

A

*

39
Q

A patient comes to see you with very high blood pressure and a fast heart rate. You hope to relieve the stress on his cardiovascular system by administering bethanecol. How does administration of this drug affect the peripheral vascular resistance in this patient?

A

Although there is no direct parasympathetic innervation on the blood vessels, they do have M3 receptors. Bethanecol is a muscarinic agonist and will stimulate these M3 receptors, causing activation of guanylyl cyclase, production of NO and vasodilation.

40
Q

A 22 year old female is at the gym and has a sudden asthma attack. What adrenergic drug could you give her that would help dilate her bronchioles and what receptor does this drug act on?

A

Albuterol. Beta-2 agonist.

41
Q

A patient comes to the ED in cardiogenic shock. What adrenergic drug could you give him that would increase perfusion of the kidneys, improve heart contractility, increase heart rate and cardiac output? What receptors does this drug act on?

A

Dopamine. D1 and Beta-1 agonist.

42
Q

A patient comes to see you with a constant runny nose. What adrenergic drug could you prescribe for his runny nose and what receptors does it act on?

A

Phenylephrine. Alpha-1 and a slight alpha-2 agonist.

43
Q

A 44 year old male comes to see you with hypertension due to increased peripheral resistance. What adrenergic drug could you prescribe to help control his blood pressure and what receptors does it act on?

A

Prazosin. Alpha-1 antagonist.

44
Q

A 58 year old male comes to see you with symptoms consistent with angina pectoris. What adrenergic drug could you prescribe him and what receptors does this drug work on?

A

Propanolol. Beta-1 and beta-2 antagonist

45
Q

A 24 year old female comes to see you with a new history of supra ventricular arrhythmias. What adrenergic drug could you prescribe her and what receptors does it work on?

A

Metoprolol. Beta-1 antagonist.

46
Q

A 55 year old male with benign prostatic hyperplasia comes to see you. What adrenergic drug could you prescribe him and what receptors does it work on?

A

Tamsulosin. Alpha-1A selective antagonist.

47
Q

A 33 year old female was diagnosed with pheochromocytoma-related hypertension. What adrenergic drug could you prescribe to get her hypertension under control and what receptors does it act on?

A

Cavedilol. Alpha-1, Beta-1 and Beta-2 antagonist.

48
Q

Epinephrine stimulates renin release from the juxtaglomerular cells of the kidney by activating which type of adrenergic receptor?

A

Beta-1

49
Q

From the graph below, drug X is most likely:

A

Epinephrine. Note the increase in systolic BP, HR (beta-1, alpha-1) and decrease in diastolic BP (beta-2).

50
Q

From the graph below, drug X is most likely:

A

Prazosin. Note the increased diastolic pressure from blocking alpha-1 receptors.

51
Q

Due to the vasodilating properties of hydrazine and minoxidil, reflex tachycardia is a common side effect from hypotension. What can you prescribe to prevent this reflex tachycardia?

A

Beta-blocker

52
Q

What vasodilating drugs tend to act mostly on arterioles? What ones act primarily on venules?

A

Arterioles = minoxidil and hydralazine. Venules = organic nitrates

53
Q

How do the two different classes of calcium channel blockers compare in the categories listed below?

A

*

54
Q

A patient comes to see you with supra ventricular tachycardia. You are very concerned about conduction of signals to the ventricles and causing ventricular tachycardia. What calcium channel blockers would you use for this patient?

A

Non-dihydropyridines (verapamil or diltiazem)

55
Q

What calcium channel blockers are most effective at reducing after load and are useful in elderly and African-American populations?

A

Dihydropyridines (nifedipine and nicardipine)

56
Q

A 41 year old male was prescribed a medication for his high blood pressure. He goes back to his doctor and complains of onset of chronic cough since he has been on the medication. What type of medication was he likely prescribed for hypertension and how is this causing his cough? What drug could you prescribe as an alternative to this patient?

A

He was probably prescribed an ACE inhibitor (anything ending in “pril”). ACE inhibitors also inhibit the breakdown of bradykinin, which causes bronchial constriction. You could prescribe Losartan (and ARB which inhibits the AT1 receptor)

57
Q

A patient comes to see you a few weeks after receiving a medication for hypertension. He has been feeling very ill and lab results show hyperkalemia. Name two drugs that may have been prescribed to him that can cause hyperkalemia.

A

Spironolactone or eplerenone. These are both mineralcorticoid antagonists (increasing Na+ and H2O excretion while retaining more K+). These drugs essentially reverse the effects of aldosterone.

58
Q

A patient comes to see you with high blood pressure. She has high lab values of renin in her system and you give her drugs to target that system. What contraindications should you have considered before prescribing this medication?

A

Pregnancy, renal artery stenosis and hyperkalemia. These are all contraindicated for drugs that target the renin, angiotensin and aldosterone systems.

59
Q

A patient comes to see you complaining of chest pain while he exercises that goes away when he stops. History is only remarkable for hypertension. What classes of drugs could help relieve his chest pain when he exercises?

A

Angina is a consequence of increased preload (during exercise), increased after load (hypertension) and increased heart rate (exercise). All of these factors either increase the O2 usage in the heart or decrease diastolic filling time (and thus decrease time for coronary perfusion). Organic nitrates dilate venules (decrease preload), Ca2+ channel blockers are negative inotropes and vasodilators (decreasing preload and after load), and beta-blockers decrease HR, contractility and allow for greater diastolic filling time.

60
Q

A patient is having chest pain and tells you he thinks he should take nitroglycerin to relieve his chest pain. What contraindications should you check before giving him nitroglycerin?

A

Has he taken sildenafil in the last 24 hours (PDE5 inhibitor, combination leads to fatal hypotension), is he hypotensive, does he have hypertrophic cardiomyopathy, and does he have diastolic heart failure.

61
Q

A 56 year old male with a history of diabetes and renal failure comes to your office for his annual check up. What is the target blood pressure for this patient?

A

For a patient with diabetes AND kidney disease, the target BP is 130/80.

62
Q

A patient comes to the ED in paroxysmal suprventricular tachycardia. You pull out the fastest acting drug you have, inject it and his symptoms go away. What drug did you use and how did it work?

A

Adenosine. This is the first line drug used for PSVT. It activates Go and Gi proteins that open K+ channels and block Ca2+ channels. This inhibits AV>SA nodal conduction.

63
Q

A 54 year old patient comes to the ED in ventricular tachycardia. He has a history of MI. What is the first choice drug for this patient?

A

Amiodarone. K+ channel blocker will slow myocyte repolarization and help control the tachycardia.

64
Q

What class of drugs is commonly used in the treatment of angina pectoris, acute MI, hypertension, CHF and arrhythmias (basically everything)?

A

Beta blockers

65
Q

A 62 year old patient with CHF comes to see you complaining of increased pulmonary congestion. You prescribe him the only inotropic drug that does not decrease mortality. What drug did you prescribe and how does it work?

A

Digoxin. It blocks the Na+/K+ ATPase. Na+ builds up and Ca2+ can no longer exit the cell via the Na+/Ca2+ exchanger and Ca2+ builds up. Digoxin also increases vagal tone which slows AV conduction.

66
Q

An 80-year old woman comes to the ED with a history of dehydration and syncope. She is currently taking digoxin and captopril for CHF and a-fib. Her blood work reveals an elevated serum potassium level and a toxic digoxin level. While you are assessing her she develops a 30 second long VT which spontaneously converts to a slow, wide complex rhythm with complete AV block. What treatment is appropriate for this woman?

A

IV Digibind Fab fragments and terminating digoxin and captopril. This will cure her of hyperkalemia.

67
Q

A 43 year old male with high cholesterol asks you if he should be on a statin. Why would a drug of this class help lower his cholesterol?

A

It is an HMG-CoA Reductase inhibitor, decreasing cholesterol synthesis in the liver. This causes the liver to express increased LDL receptors and take more cholesterol out of the blood.

68
Q

A patient comes to see you with elevated VLDL and decreased HDL levels. What drug could you prescribe her that will decrease incidence of MI and CHD by 22%?

A

Fenofibrate. These drugs not only decrease VLDL levels, but the increase ApoA-1 synthesis which results in a 10-15% increase in HDL levels. They also increase LPL activity and decrease VLDL synthesis in the liver.

69
Q

Identify which region of the heart is at risk for ischemic injury when the artery indicated below is occluded.

A

LCA (LA, LV, IVS, & AV node), LAD (LV, RV & anterior 2/3 of IVS), LCX (LA & LV), LT marginal (LV), RCA (RA, posterior IVS, SA, and AV nodes), Sinuatrial (Pulmonary trunk & SA node), RT marginal (RV & apex), posterior interventricular (RV, LV and posterior 1/3 of IVS).

70
Q

A patient comes to see you with chest pain and you diagnose him with angina. You perform an EKG on him and conclude that there is a left axis deviation with an elevate TP segment. What coronary artery blockade is causing these symptoms and where does it branch from?

A

Left axis deviation = left sided. Elevated TP segment = posterior heart. The posterior descending coronary artery supplies the posterior ventricular walls and 1/3 of the ventricular septum. This artery is usually from the RCA (85% of people) but can also branch from the LCX (8%) and come from the LCX and RCA (7%).

71
Q

A patient is in your office having an EKG run due to suspicious palpitations. The patient gets thirsty and has a cold glass of water during the EKG, which triggers irregular, tachycardic heart beats. Why did this change occur?

A

The left atrium is the most posterior chamber of the heart and actually sits right on the esophagus. Cold water to the atrial wall can trigger arrhythmias.

72
Q

You go to the gym to start working out for you PT test and hop on the treadmill without stretching before you go. You run for 30 minutes, get off the treadmill and go home. What is responsible for maintaining cardiac output in the first minutes of your run? How about in the last minutes of your run?

A

At first, cardiac output is modulated by heart rate and stroke volume. After the initial increase in cardiac output, stroke volume peaks and any additional increase in cardiac output is mitigated by heart rate.

73
Q

You are performing a surgery on a patient and want to stop to make sure her cardiac output is still at a safe level. The anesthesiologist tells you she is consuming oxygen at a rate of 200mL/min, arterial [O2] = 50mL/L of blood and venous [O2] = 30mL/L of blood. What is her cardiac output?

A

You must use the Fick equation for this. CO = rate of O2 consumption/(arterial [O2] - venous [O2]). So, CO = 200/(50-30) = 10L/min.

74
Q

A 34 year old female is hemorrhaging after a serious car accident. On the way to the hospital, the paramedic assesses the body’s ability to perfuse the organs. He notes that cardiac output is 2L/min, mean venous pressure is 2 and total peripheral resistance is 30. How soon will this patient need blood transfusion?

A

Total Peripheral Resistance = (MAP - MVP)/CO. So 30 = (MAP - 2)/2. MAP = 62. Any mean arterial pressure below 60mmHg puts you at risk for organ failure due to decreased perfusion pressure. She will need transfusion ASAP.

75
Q

How can you calculate MAP from your patient’s blood pressure reading?

A

MAP = (2/3)diastolic pressure + (1/3)systolic pressure

76
Q

A 70 year old male comes to see you complaining of orthopnea and palpitations. When auscultating his heart you hear a diastolic decrescendo murmur over the lower left sternal border. You also note water-hammer pulses when palpating his carotids. How does this patient’s condition affect the stroke volume of his heart?

A

Aortic regurgitation results in a large increase in pulse pressure. Pulse pressure is inversely related to stroke volume and, consequently, stroke volume is decreased.

77
Q

A 91 year old male with CHF comes to your clinic complaining of orthopnea and increasing dyspnea. Echocardiography reveals decreased stroke volume. What things could you modify to help this patient increase stroke volume and clear fluid from his lungs? What things do you NOT want to give this patient?

A

“CAP”. Stroke volume increases with increased CONTRACTILITY (increase intracellular Ca2+ or decrease extracellular Na+), decreased AFTERLOAD and increased PRELOAD. You do not want to give this patient things that decrease stroke volume like beta-blockers (decreases cAMP and contractility) or non-dihydropyridine Ca channel blockers like Verapamil (decreases contractility).

78
Q

If point A on this curve is you sitting in a chair, how would exercise change where the you sit on the curve?

A

Point C. Exercise causes an increase in preload from sympathetic stimulated venoconstriction. Increased preload results in greater contractility and a higher stroke volume

79
Q

When assessing one of your patient for left-sided heart failure, you determine his end systolic volume to be 10mL and his end diastolic volume to be 40mL. Is this patient above the cutoff for preserved left ventricular function?

A

Ejection fraction = (EDV - ESV)/EDV. (30-10)/30 = 66%. Normal ejection fraction = 55%, so your patient is A okay.

80
Q

A 43 year old male with adenocarcinoma of the liver comes to your clinic in a hypercoaguable state. You are considering putting him on warfarin to thin his blood and want to know what the current resistance to flow in his arteries are for dosing purposes. What factors will contribute to increased resistance to flow in this patient?

A

Increased pressure, viscosity and length. Decreased flow and radius.

81
Q

Below is a diagram indicating how different organs are connected by blood flow in the body. Which vessels will experience the greatest amount of resistance to flow between one another? Which will experience the least amount of resistance to flow?

A

Total resistance is always the lowest in vessels that run in parallel because you add the reciprocals of each resistance to each other, then the reciprocal of your total is the total resistance of the vessels in parallel. In vessels that run in series, you just add each resistance to each other.

82
Q

What is the target hematocrit level for men and women?

A

40-45%. Higher = polycythemia and increased viscosity. Lower = anemia.

83
Q

What point on this graph represents mean systemic filling pressure

A

Mean systemic filling pressure = x-intercept, this is where CO = 0 because right atrial pressure is high enough to stop blood flow. Exercise will increase the cardiac output curve, requiring blood flow to catch up. CHF results in a decrease in CO.

84
Q

What points on the graph below represent isovolumetric contraction, systolic ejection, isovolumetric relaxation, rapid filling and reduced filling, S1, S2, S3 and S4?

A

1 = isovolumetric contraction, 2 = systolic ejection, 3= isovolumetric relaxation, 4= rapid filling and 5 = reduced filling

85
Q

What points on the Wigger’s diagram below represent S1, S2, S3, S4, a wave, c wave, x descent, v wave, y descent, aortic valve opening, mitral valve opening, atrial systole, isovolumic contraction, ejection, isovolumic relaxation, and ventricular filling phase.

A

1 = a wave, 2 = c wave, 3 = x descent, 4 = v wave, 5 = y descent, 6 = aortic valve opening, 7 = mitral valve opening, green = atrial systole, orange = isovolumic contraction, yellow = ejection, pink = isovolumic relaxation, and blue = ventricular filling phase

86
Q

A 22 year old male comes to see you in clinic. While listening to his heart you hear a nice P2 split on inspiration that goes away with exhalation. Why does the P2 come out during inhalation?

A

Intrathoracic pressure drops when he inhales. This increases venous return to the right ventricle, increasing RV stroke volume and delaying closure of the pulmonic valve.

87
Q

A 33 year old female comes in for her yearly check-up. When listening to her heart sounds you notice widened P2 and A2 splitting that does not change with inspiration. What conditions could contribute to this finding?

A

Anything that delays RV emptying (pulmonic stenosis, right bundle branch block, increased preload from ASD) or accelerates LV emptying (mitral valve stenosis).

88
Q

A 42 year old male comes to see you for his first ever doctor’s visit. When listening to his heart you note an A2, P2 split that disappears with inspiration. What contributes to this condition?

A

This is paradoxical splitting of the second heart sounds. In conditions that delay LV emptying (aortic stenosis, left bundle branch block) the P2 sound precedes the A2 sound and causes splitting. On inspiration, P2 occurs later and the splitting goes away as it moves closer to the delayed A2 sound.

89
Q

Where do you auscultate for suspected systolic murmurs (aortic stenosis, mitral regurgitation, HOCM, pulmonic stenosis, PDA), pansystolic murmurs (tricuspid regurgitation, VSD) and diastolic murmurs (aortic regurgitation, pulmonic regurgitation, tricuspid stenosis, and mitral stenosis)?

A

1 = Aortic stenosis. 2 = Pulmonic stenosis. 3 = HOCM, aortic regurgitation, pulmonic regurgitation. 4 = Tricuspid regurgitation, tricuspid stenosis, VSD. 5 = Mitral regurgitation, mitral stenosis. 6 = Patent ductus arteriosus.

90
Q

When the foramen ovale fails to close, you get left to right shunting of blood in the heart. Since the atria are low pressure chambers, that shunting does not carry a murmur. How might you auscultate to determine if a patient has an atrial septal defect?

A

Auscultate the pulmonic and tricuspid for flow rumbles during systole and diastole respectively. These rumbles are due to the increased pressure in the right side of the heart.

91
Q

A 65 year old male comes to see you for a check-up. When auscultating his heart you hear a holosystolic, high-pitched blowing murmur, best heard at the apex of the heart which also radiates towards the axilla. What maneuvers might you have to patient do to confirm the most likely diagnosis?

A

This is mitral valve regurgitation. You can have him do things that increase left heart volume (exhale) or TPR (hand-grip or squat). If the murmur gets louder, you have provided further evidence of mitral valve regurgitation.

92
Q

A 65 year old male comes to see you for a check-up. When auscultating his heart you hear a holosystolic, high-pitched blowing murmur, best heard at the tricuspid area, which also radiates to the right sternal border. What maneuvers might you have to patient do to confirm the most likely diagnosis?

A

This is tricuspid valve regurgitation. You can have him do things that increase right heart volume (inspiration). If the murmur gets loud you have provided further evidence of tricuspid valve regurgitation.

93
Q

A 70 year old female comes to see you in the cardiology clinic. When auscultating around the aortic area you hear a crescendo-decrescendo ejection murmur followed by a click. You note that the murmur radiates to the carotids where pulses are weak and delayed. What maneuvers might you have the patient do to confirm the most likely diagnosis?

A

Squat (increased afterload) or Valsalva (decreased preload) decrease the amount of blood in the left heart and these maneuvers will make the ejection murmur weaker. This will further strengthen you suspicion of aortic stenosis.

94
Q

A 24 year old male comes to your clinic for a military physical exam. When auscultating his heart you hear a harsh holosystolic murmur which is loudest at the tricuspid area. You think he may have ventricular septal defect. What maneuver can you have him do to aid you in your diagnosis?

A

Handgrip or squat maneuver. This increases afterload, increasing blood in the left heart that will exaggerate the murmur as more blood passes from the left ventricle to the right ventricle during systole.

95
Q

A 42 year old male comes to your office for a routine check up. Upon auscultation, you hear a late systolic crescendo murmur with a mid systolic click over the apex of the heart. What maneuver can you have the patient do to further your narrowing of this diagnosis?

A

This is mitral valve prolapse. Having the patient do the Valsalva maneuver decreases venous return, decreasing blood volume in the left heart which will make the prolapse worse and the murmur will become more audible.

96
Q

A 65 year old female comes in for a yearly check up. When auscultating her heart you note an early, high-pitched, blowing diastolic decrescendo murmur at the left sternal border. You also note that her head bobs with each pulse. What maneuver can you have this patient do to further narrow the diagnosis?

A

This patient has aortic regurgitation. The handgrip or squat test because increasing afterload will increase the amount of blood that back flows through the aorta, making the murmur louder and further narrowing your diagnosis.

97
Q

A 78 year old male comes to see you for a check up. Auscultation of his heart yields an opening snap at the mitral area. What maneuver can you perform to narrow your diagnosis of this patient?

A

This is mitral stenosis. Having the patient exhale will increase left atrial return, increasing the blood that flows through the stenosed valve. This will increase the intensity of the murmur and further narrow your diagnosis.

98
Q

A 10 year old female comes to see you for a wellness check up. When you auscultate her heart you note a continuous machine-like murmur that is loudest right after S2. Where can you auscultate to achieve the best position for hearing this murmur?

A

Left infraclavicular area. This is a murmur from a patent ductus arteriosus.

99
Q

What are the different phases and ion conductances of the action potential seen below? Where do these types of action potentials occur? When does muscle contraction occur?

A

This is a diagram of a ventricular action potential (which also occurs in the His bundle and purkinje fibers). Muscle contraction occurs when VG Ca2+ channels open and cause calcium-induced calcium release from the sarcoplasmic reticulum in cardiac myocytes.

100
Q

What are the different phases of ion conductances of the action potential seen below? Where do these action potentials occur? How does sympathetic stimulation affect this action potential?

A

This is a diagram of a nodal action potential (chick occurs in the AV and SA nodes). Sympathetic stimulation increases the rate of diastolic depolarization by increasing the amount of open funny Na+ channels.

101
Q

When analyzing a patient’s ECG, what is a normal PR interval? QRS complex?

A

PR < 200 msec. QRS < 120 msec.

102
Q

What is represented by the PR interval?

A

Impulse conduction delay through the AV node.

103
Q

What is represented by the QT interval?

A

Mechanical contraction of the ventricles.

104
Q

What is indicated by an inverted T wave?

A

Recent MI

105
Q

At what point in the ECG are all ventricles depolarized?

A

ST segment

106
Q

What cardiac structures are best at signal conduction?

A

Purkinje > Atria > Ventricles > AV node

107
Q

What cardiac structures are the best pacemakers?

A

SA > AV > His > Purkinje

108
Q

How much of a delay from the SA node do the ventricles need for filling before they depolarize?

A

100 msec, this is done by the AV node.

109
Q

A patient presents to the ER with palpitations. His ECG is shown below. What is your diagnosis and what do you give him to try and prevent ventricular fibrillation?

A

This is Torsades de Pointes, a resultant arrhythmia from a patient with long QT syndrome. You need to give him magnesium sulfate to regain control of the intracellular pumps that allow for ion balance in the heart (K+, Na+ and Ca2+).

110
Q

A patient presents to the ER with palpitations. His ECG is shown below. What is your diagnosis and what do you give for treatment?

A

This is atrial fibrillation. Note the chaotic and irregular baseline. Note absence of p-waves between QRS complexes. Atrial fibrillation can lead to stasis and stroke so you would give this patient anticoagulants and possible cardioversion to restore normal rhythm.

111
Q

A patient presents to the ER with palpitations. His ECG is shown below. What is your diagnosis and what do you give for treatment?

A

This patient has atrial flutter. Note back-to-back “sawtooth” appearance of p-waves that indicate repetitive atrial depolarizations. You would give antiarrhythmics (IA, IC or III) to restore sinus rhythm. You would also give beta-blockers or Ca-channel blockers to control HR.

112
Q

A patient presents to the ER with palpitations. His ECG is shown below. What is your diagnosis and what do you give for treatment?

A

This is ventricular fibrillation. Note the rhythm is completely erratic with no identifiable waves. This is fatal with out CPR or defibrillation.

113
Q

You are practicing ECGs on your fellow medical student friend. His ECG is shown below. What is your diagnosis and what do you give for treatment?

A

This patient has 1st degree AV block. Note prolonged PR interval. This is an asymptomatic condition.

114
Q

You are practicing ECG readings on your medical student buddy. His ECG is shown below. What is your diagnosis and what do you give for treatment?

A

This patient has 2nd degree AV block (Mobitz type I or Wenckebach). Note progressive lengthening of PR interval until a p-wave is dropped. This is normally an asymptomatic condition.

115
Q

A patient presents to the ER with palpitations. His ECG is shown below. What is your diagnosis and what do you give for treatment?

A

This is a 2nd degree AV block (Mobitz II). Note that the dropped beats are not preceded by a lengthening PR interval. Often there will be 2 P waves to 1 QRS wave. This condition needs to be treated with a pacemaker.

116
Q

A patient presents to the ER with palpitations and history of Lyme disease.. His ECG is shown below. What is your diagnosis and what do you give for treatment?

A

This is a 3rd degree AV block (complete). 3rd degree block occurs in patients with Lyme disease. Notice how the atria and ventricles beat independently of each other. P waves and QRS complexes are present, but have no relation to one another. This is usually treated with a pacemaker.

117
Q

After having surgery to replace his tricuspid valve your patient has experienced a number of complications. One being increased edema, blood pressure and orthopnea. How might surgical damage cause the symptoms experienced by this patient?

A

Damage to myocytes in the right atrium could decrease their ability to sense pressure changes and secrete atrial natriuretic peptide. Normally, this peptide constricts efferent renal arterioles and dilates afferent arterioles. This results in diuresis, excretion of Na+ and conservation of K+.

118
Q

Back in the day, surgeons used to cut the vagus nerve in patients who had terrible peptic ulcers. How would ligation of the vagus nerve affect systemic control of blood pressure? What other nerve plays a role in systemic BP control?

A

The aortic arch transmits signals to the solitary nucleus of medulla when BP is high via the vagus nerve. Ligation of this nerve would obviously decrease the brain’s ability to sense higher blood pressures. Luckily, the carotid sinus also has the glossopharyngeal nerve (sinus nerve) that transmits signals to the solitary nucleus of medulla with any change in blood pressure.

119
Q

A 31 year old soldier lost both lower limbs in an IED blast and is hemorrhaging. He lost a large amount of blood, but luckily tourniquets were applied and allowed his body to take control. What role does the carotid sinus play in restoring systemic blood pressure in this soldier?

A

Hypotension from blood loss -> decreased carotid stretch -> decreased afferent signal firing -> increased efferent sympathetic firing and decreased efferent parasympathetic firing -> vasoconstriction, increased HR, increased contractility and ultimately an increased blood pressure.

120
Q

A patient is in the hospital after a car accident and is recovering from sever hemorrhage. Why might this not be the best time to check for his carotid pulse?

A

Carotid massage -> carotid stretch -> afferent signal firing -> decreased sympathetic efferent signal and increased parasympathetic efferent signal -> decreased HR!

121
Q

A patient comes to the ED after slamming his head into a tree while snowboarding. Vital signs are BP 180/110, HR 45 and RR 10. Considering the patient hit his head, what is causing his vitals to present this way?

A

Cushing reaction (the triad of hypertension, bradycardia and respiratory depression). This is caused by increased intracranial pressure from hemorrhage that constricts arterioles. Constricted arterioles causes cerebral ischemia, which elicits sympathetic reflex to increase perfusion pressure (hypertension). This then increases carotid stretch which stimulates the parasympathetic system to induce bradycardia.

122
Q

What metabolites in the blood stimulate the carotid and aortic bodies to increase blood pressure?

A

Decreased PO2 (<60mmHg), increased PCO2 and decreased blood pH.

123
Q

What blood metabolites affect the chemoreceptors that govern systemic blood pressure in the CNS?

A

Only arterial PCO2 has an effect on the brain. This alters pH and PCO2 in the brain’s ICF.

124
Q

Laceration of what organ in a sword fight would potentially empty the most blood out of your opponent?

A

The lungs. They receive 100% of cardiac output.

125
Q

Why does ischemia always follow if myocardial O2 demand cannot be met by coronary vasodilation?

A

Myocardial cells already take out 80% of the O2 carried by Hgb. There is no wiggle room for increased O2 extraction from blood, only increased flow.

126
Q

You are evaluating a patient with symptoms of CHF with the Swan-Ganz catheter. You get to the pulmonary capillaries and wedge you catheter and find that pulmonary capillary wedge pressure is greater than left ventricular diastolic pressure. Is this a normal finding?

A

No. PCWP should be around left atrial pressure which should be around diastolic ventricular pressure. This finding indicates aortic stenosis, which increases left atrial pressure to a greater value than ventricular diastolic pressure.

127
Q

You decide to go out for a morning run because your PT test is coming up soon. You start with a slow jog to warm up and your body starts to increase blood flow to the heart as your heart rate rises. How is this increase in blood flow to the heart controlled? How is increased blood flow to the muscle achieved when you are running?

A

CO2, NO and adenosine mediate coronary vasodilation. It makes sense because as you exercise you produce CO2 and adenosine which act to vasodilate the coronary vessels. Increased flow to the muscle occurs as you produce lactate, adenosine and K+, which all act as vasodilators.

128
Q

How do the kidneys auto regulate blood flow?

A

Myogenic. The glomerulus is very sensitive to changes in blood flow and vasoconstricts with increased flow and vasodilates with decreased flow. This allows for a constant perfusion pressure.

129
Q

You are hiking Mt. Everest and forgot your oxygen mask. As you go up in elevation you notice your fingers get fatter and it gets harder to breath. What principle of auto regulation is causing these symptoms?

A

Hypoxia in the lungs causes vasoconstriction where in the rest of the body it causes vasodilation.

130
Q

A 33 year old female with metastasized breast cancer comes to your office concerned about edema in her legs. If her hydrostatic capillary pressure is 20, hydrostatic interstitial pressure is 10, osmotic capillary pressure is 5 and osmotic interstitial pressure is 20, where is the net filtration pressure in her capillaries?

A

Pnet = [(Pc - Pi) - (Oc - Oi)]. Pnet = (20-10) - (5-20) = 25.

131
Q

How do you calculate net fluid flow from the capillary bed to the interstitial space?

A

Jv (net fluid flow) = Kf (capillary permeability) * Net filtration pressure.

132
Q

A patient comes to your office with edema in his legs. If this was the only information you have from the visit because he is Chinese and can’t speak English, what is your differential diagnosis?

A

Increased capillary hydrostatic pressure from CHF, decreased capillary osmotic pressure from liver failure or nephrotic syndrome, increased capillary permeability from burns, toxins or infections, or increased interstitial osmotic pressure from blocked lymphatic drainage.

133
Q

A patient comes to see you with a systolic murmur heard at the tricuspid area. Echocardiography reveals that she has Tetralogy of Fallot. She says for years she has squatted down whenever she felt short of breath and it relieved her symptoms. How does that work?

A

Squatting down increases after load. Increased after load increases pressure in the left heart, decreasing the amount of deoxygenated blood shunted over from the right heart.

134
Q

You are an OB/GYN and it is imperative that you don’t switching these two medications around when a mother gives birth to a baby with a patent ductus arteriosus?

A

If you need to keep the PDA open, use PGE. If you need to close it, use indomethacin.

135
Q

A diabetic mother comes to your office because she is pregnant. She asks if her diabetes will affect her baby’s heart development at all. What congenital birth defect is associated with maternal diabetes?

A

Transposition of the great vessels

136
Q

A young child is born with congenital rubella. After stabilizing the child, what cardiac anatomical structures do you need to analyze?

A

Septal defects, PDA and pulmonary stenosis are all associated with congenital rubella.

137
Q

What congenital heart defects are associated with 22q11 syndromes?

A

Truncus arteriosus and tetralogy of Fallot.

138
Q

A 22 year old male comes to see you complaining of chest pain. He has a blood pressure of 150/90 and you are concerned to see this in someone his age. He says his family has some sort of cholesterol disease. What physical exam findings would indicate a hyperlipidemia?

A

Xanthomas and corneal arcus (lipid on the cornea).

139
Q

What are the three different types of arteriosclerosis?

A

Monckeberg (media calcification), arteriolosclerosis (hyaline and hyperplastic) and Atherosclerosis (plaques and atheromas).

140
Q

What is the most common location in the body you will find atherosclerotic plaques?

A

Abdominal aorta.

141
Q

A 24 year old male presents to your clinic with chest pain. While running his ECG, you notice a run of elevated ST segments that go back to normal after 30 seconds. What type of angina is this?

A

Prinzmetal’s variant angina. The other anginas exhibit a depressed ST segment.

142
Q

A patient comes to the ED with crushing chest pain, diaphoresis, nausea and vomiting. You hook him up to an ECG to see if he is having an MI because that is the gold standing in the first 6 hours of an MI. What do you expect to see if the patient has had a transmural infarct? A subendocardial infarct?

A

Transmural = elevated ST segment. Subendocardial = depressed ST segment.

143
Q

A patient comes to the ED with crushing chest pain, diaphoresis, nausea and vomiting. You hook him up to an ECG to see if he is having an MI because that is the gold standing in the first 6 hours of an MI. Indeed, he is having an MI, your next task is to locate it. How do you know if the infarct location is in the anterior wall (LAD)? Anterior septum (LAD)? Anterolateral wall (LCX)? Lateral wall (LCX)? Inferior wall (RCA)?

A

Look for Q waves in particular leads. Anterior wall (LAD) = V1-V4. Anterior septum (LAD) = V1 and V2. Anterolateral wall (LCX)= V4-V6. Lateral wall (LCX) = I, aVL. Inferior wall (RCA) = II, III, aVF.

144
Q

A 49 year old male comes to see you several weeks after MI complaining of chest pain again. Upon auscultation you hear a friction rub. What is your diagnosis?

A

Dressler’s syndrome. This happens when myocardium is exposed during the MI and the body develops antibodies against it, attacking it, and causing fibrinous pericarditis.

145
Q

What medications do you use to treat dilated cardiomyopathy?

A

ACE-I. Diuretics. Digoxin.

146
Q

What medications do you use to treat HOCM?

A

Beta-blockers, non-dihydropyridine calcium channel blockers.

147
Q

What drugs have been proven to decrease mortality in patients with CHF?

A

ACE-I, beta-blockers, ARBs, and spironolactone.

148
Q

A patient you are treating has severely distended neck veins, decreased heart sounds, increased HR and hypotension. How could you use this patient’s blood pressure to narrow down you differential?

A

Look for pulsus paradoxus (10mmHg + drop in systolic BP with inspiration). This is typical of cardiac tamponade.

149
Q

Why are patients who catch some syphilis at risk for aortic aneurism?

A

Tertiary syphilis disrupts the vasa vasorum of the aorta and causes aortic atrophy and dilation.

Decks in CPR I Class (48):